LSAT and Law School Admissions Forum

Get expert LSAT preparation and law school admissions advice from PowerScore Test Preparation.

 Administrator
PowerScore Staff
  • PowerScore Staff
  • Posts: 8916
  • Joined: Feb 02, 2011
|
#66084
Please post your questions below!
User avatar
 christinecwt
  • Posts: 74
  • Joined: May 09, 2022
|
#95653
Hi Team - Sorry that do you mind explaining this question and stimulus? Also, I don't understand why Answer Choice D is correct as I picked Answer Choice B? Many thanks! :-D :-D
User avatar
 katehos
PowerScore Staff
  • PowerScore Staff
  • Posts: 184
  • Joined: Mar 31, 2022
|
#95697
Hi Christine!

Let's start with the stimulus. In the first sentence, we see the columnist's conclusion: we should not reroute high-tension power lines away from highly populated areas. Then, we find some of the columnist's rationale in the second sentence, where the columnist explains that limited resources should be devoted to protecting the public from only well-substantiated public health threats. After reading this, we can sense that something isn't lining up. How did we get from only devoting limited resources to well-substantiated public health threats to not rerouting power lines? The columnist took a bit of a leap!

When we get to the question stem, it's a relief! This is a Justify question, so it makes sense that there was something missing from the stimulus -- we're going to look for that missing link in the answer choices. Since this is a Justify question, we can use the Justify Formula: premises + answer choice = conclusion.

With this in mind, let's prephrase. Consider what missing link could heal that gap we noticed earlier between not rerouting power lines and the assertion that resources should only be used to protect against substantiated threats? Well, if we know that high-tension power lines are not a well-substantiated public health threat, then we should not devote our resources to rerouting the power lines.

If we plug answer choice (B) into our formula, we can see that it does not justify the reasoning in the stimulus - it's irrelevant!
      Devote Resources :arrow: Well-substantiated Threat + Proponents Ignore Economic Realities /= We Should Not Reroute Power Lines

Since we are now able to eliminate (B), let's take a look at (D). When we plug (D) into our formula (note that we are using the contrapostitive) we can see that it works!
      Not a Well-Substantiated Threat :arrow: Not Devote Resources + Power Lines are Not a Threat = We Should Not Reroute Power Lines.

I hope this helps! :)
Kate

Get the most out of your LSAT Prep Plus subscription.

Analyze and track your performance with our Testing and Analytics Package.